-
Notifications
You must be signed in to change notification settings - Fork 0
/
Copy pathhw05.tex
379 lines (329 loc) · 24.5 KB
/
hw05.tex
1
2
3
4
5
6
7
8
9
10
11
12
13
14
15
16
17
18
19
20
21
22
23
24
25
26
27
28
29
30
31
32
33
34
35
36
37
38
39
40
41
42
43
44
45
46
47
48
49
50
51
52
53
54
55
56
57
58
59
60
61
62
63
64
65
66
67
68
69
70
71
72
73
74
75
76
77
78
79
80
81
82
83
84
85
86
87
88
89
90
91
92
93
94
95
96
97
98
99
100
101
102
103
104
105
106
107
108
109
110
111
112
113
114
115
116
117
118
119
120
121
122
123
124
125
126
127
128
129
130
131
132
133
134
135
136
137
138
139
140
141
142
143
144
145
146
147
148
149
150
151
152
153
154
155
156
157
158
159
160
161
162
163
164
165
166
167
168
169
170
171
172
173
174
175
176
177
178
179
180
181
182
183
184
185
186
187
188
189
190
191
192
193
194
195
196
197
198
199
200
201
202
203
204
205
206
207
208
209
210
211
212
213
214
215
216
217
218
219
220
221
222
223
224
225
226
227
228
229
230
231
232
233
234
235
236
237
238
239
240
241
242
243
244
245
246
247
248
249
250
251
252
253
254
255
256
257
258
259
260
261
262
263
264
265
266
267
268
269
270
271
272
273
274
275
276
277
278
279
280
281
282
283
284
285
286
287
288
289
290
291
292
293
294
295
296
297
298
299
300
301
302
303
304
305
306
307
308
309
310
311
312
313
314
315
316
317
318
319
320
321
322
323
324
325
326
327
328
329
330
331
332
333
334
335
336
337
338
339
340
341
342
343
344
345
346
347
348
349
350
351
352
353
354
355
356
357
358
359
360
361
362
363
364
365
366
367
368
369
370
371
372
373
374
375
376
377
378
379
\documentclass[
pages,
boxes,
color=WildStrawberry
]{homework}
\usepackage{cleveref}
\usepackage{macros}
\usepackage{qtree}
\usepackage{booktabs}
\input{config}
\hwnum{5}
\duedate{Wed, Mar 24, 2020 10:00 PM}
\colorlet{Ppink}{MidnightBlue!20}
\newtcbox{\mafbox}[1][]{on line, math upper,
boxsep=4pt, left=0pt,right=0pt,top=0pt,bottom=0pt,
colframe=white,colback=Ppink,
highlight math style={enhanced}
}
\begin{document}
\begin{problem}
Consider the adjoint representation of $\slie{3}{\C}$ as a representation of $\slie{2}{\C}$ by restriction to the subalgebra $\mathsf{g}_1 = \vspan_{\C}\{H_1, X_1, Y_1\} \simeq \slie{2}{\C}$.
\begin{parts}
\part{Decompose this representation as a direct sum of irreducible representations of $\slie{2}{\C}$.}\label{part:1a}
\part{Which isomorphism types appear in the decomposition in (a), and with what multiplicity?}\label{part:1b}
\end{parts}
\end{problem}
\begin{solution}
\ref{part:1a}
Since the wording of this question is quite confusing, it's helpful to clarify how I interpreted the question. We're working with the representation $\ad\!|_{\mathsf{g}_1}:\mathsf{g}_1\to\mathsf{gl}(\slie{3}{\C}) = \End(\slie{3}{\C})$.
In order to understand the invariant subspaces of $\ad\!|_{\mathsf{g}_1}$ we first find the eigenvectors of $\ad_{H_1}$ that are annihilated by $\ad_{X_1}$. Indeed the commutation relations easily show we have
\begin{align*}
\comm{H_1}{X_1} & = 2X_2 & \comm{X_1}{X_1} & = 0 \\
\comm{H_1}{Y_2} & = Y_2 & \comm{X_1}{Y_2} & = 0 \\
\comm{H_1}{X_3} & = X_3 & \comm{X_1}{X_3} & = 0
\end{align*}
Now we can apply $\ad_{Y_1}$ to each one of these eigenvectors to better understand the invariant subspaces. I've ignored constants in the following chains for simplicity.
\begin{align*}
X_1 & \xrightarrow{\comm{Y_1}{X_1}} H_1 \xrightarrow{\comm{Y_1}{H_1}}Y_1 \xrightarrow{\comm{Y_1}{Y_1}} 0 \\
Y_2 & \xrightarrow{\comm{Y_1}{Y_2}} Y_3 \xrightarrow{\comm{Y_1}{Y_3}} 0 \\
X_3 & \xrightarrow{\comm{Y_1}{X_3}} X_2 \xrightarrow{\comm{Y_1}{X_2}} 0 \\
\end{align*}
Hence we have found 3 invariant subspaces.
\ref{part:1b}
Again here is where the wording is very confusing: are we talking about the adjoint representation as a whole, or simply the restriction? All the classmates I talked to thought it was the whole thing. I'll do the whole thing so I don't get points taken off for doing something that wasn't quite asked for, but maybe it was???
Since the adjoint representation is 8 dimensional, and above we found 7, we need one more. Above we never got the vector $H_2$ so we'll be looking for that. Inspecting the following two commutation relations helps us spot the last:
\begin{align*}
\ad_{Y_1}(H_1) = 2Y_1 & & \ad_{Y_1}(H_2) = -Y_2.
\end{align*}
Thus the last invariant subspace is spanned by $H_1 + 2H_2$. In sum we have
\begin{equation*}
(\ad, \slie{3}{\C})\cong (\pi_2, V_2(\C^2))\oplus (\pi_1, V_1(\C^2)) \oplus (\pi_1, V_1(\C^2)) \oplus (\pi_0, V_0(\C^2))
\end{equation*}
And so the multiplicity of 2 is 1, 1 is 2 and 0 is 1.
\end{solution}
\begin{problem}
Recall how we constructed an irreducible complex $\slie{3}{\C}$ representation with highest weight $(1, 1)$ by considering the tensor product representation $\C^3 \otimes (\C^3)^*$.
\begin{parts}
\part{Use the same method to construct an irreducible complex $\slie{3}{\C}$-representation with highest weight $(2, 0)$, acting on a subspace of $\C^3 \otimes \C^3$.}\label{part:2a}
\part{Determine the dimension of this representation, along with all the weights and their multiplicities. (The multiplicity of a weight is the dimension of its weight space.)}\label{part:2b}
\part{Decompose $\C^3 \otimes \C^3$, the tensor product of two copies of the standard $\slie{3}{\C}$-representation, into a direct sum of irreducible representations.}\label{part:2c}
\end{parts}
\end{problem}
\begin{solution}
\ref{part:2a}
Take the product basis of $\C^3\otimes\C^3$, that is $e_i\otimes e_j$ for $i,j\in\qty{1,2,3}$. As a guess we will take $e_1\otimes e_1$ as the starting point to apply $\pi_{2,0}(Y_1)$ and $\pi_{2, 0}(Y_2)$ repeatedly. Branching left indicates $Y_1$ has been applied, and right indicates $Y_2$.
\Tree[.$e_1\otimes e_1$ [.${e_1\otimes e_2 + e_2\otimes e_1}$
[.${e_2\otimes e_2}$ [.0 ]
[.${e_2\otimes e_3 + e_3\otimes e_2}$ [.0 ]
[.${e_3\otimes e_3}$ [.0 ] [.0 ]]]]
[.${e_1\otimes e_3 + e_3\otimes e_1}$
[.${e_2\otimes e_3 + e_3\otimes e_2}$ ] [.0 ]]]
[.0 ]]
Thus we have a 6 dimensional representation spanned by the symmetric vectors of $\C^3\otimes\C^3$. We can also find the associated weights by adding together the weights of $e_i$ from the standard representation.
\begin{table}[h]
\centering\begin{tabular}{r c c}
Eigenvector & Weight & Multiplicity \\ \toprule
$e_1\otimes e_1$ & $(2, 0)$ & 1 \\
$e_2\otimes e_2$ & $(-2, 2)$ & 1 \\
$e_3\otimes e_3$ & $(0, -2)$ & 1 \\
$e_1\otimes e_2 + e_2\otimes e_1$ & $(0, 1)$ & 1 \\
$e_1\otimes e_3 + e_3\otimes e_1$ & $(1, -1)$ & 1 \\
$e_2\otimes e_3 + e_3\otimes e_2$ & $(1, 0)$ & 1
\end{tabular}
\caption{Weights of product representation}\label{tab:prodweights}
\end{table}
Inspecting \cref{tab:prodweights}, and calculating $\mu_i - \mu_j = a\alpha_1 + b\alpha_2$ we can (tediously) verify that $(2, 0)$ is indeed the highest weight.
\ref{part:2b}
Refer to \cref{tab:prodweights}.
\ref{part:2c}
Since the above representation is 6 dimensional, and $\C^3\otimes \C^3$ is 9 dimensional, we need to try and find a representation that lives on the other 3 dimensions. First, note that the ``other'' 3 dimensions are spanned by
\begin{align*}
e_2\otimes e_1 - e_1\otimes e_2 & & e_3\otimes e_1 - e_1\otimes e_3 & & e_3\otimes e_2 - e_2\otimes e_3
\end{align*}
which are the antisymmetric subspace of $\C^3\otimes \C^3$.\footnote{I wish we learned about symmetric and anti-symmetric powers of vector spaces in this class, because I see them mentioned all over the place when reading material about decomposing representations.} To find what this representation looks like we can again apply $\pi_{2, 0}(Y_i)$ with the same convention as above.
\Tree[.${e_1\otimes e_2 - e_2\otimes e_1}$ [.0 ] [.${e_3\otimes e_1 - e_1\otimes e_3}$ [.${e_3\otimes e_2 - e_2\otimes e_3}$ [.0 ] [.0 ]] [.0 ]]]
This tree is exactly that of the standard representation acting on $e_1, e_2, e_3$, and hence we conclude that we have one copy of the standard representation. In final, we have
\begin{equation*}
(\pi_{2, 0}, \C^3\otimes \C^3)\cong (\pi_{1, 0}\otimes\pi_{1, 0}, \Sym^2(\C^3))\oplus (\pi_{1, 0}, \C^3).
\end{equation*}
Although I'm wondering if that last factor should be $(\pi_{1, 0}, \bigwedge^2(\C^3))$? I guess they're isomorphic, so maybe it doesn't matter? Would be good to know.
\end{solution}
\begin{problem}
Let $V_{m}(\C^3) = \vspan_{\C}\qty{z_1^k\, z_2^l\, z_3^{m - k - l} : 0 \leq k + l \leq m}$ and define $(\Pi_m(A)f)(z) = f(A^{-1}z)$ for $A \in \SU{3}$ and $f \in V_m(\C^3)$.
\begin{parts}
\part{Prove that $(\Pi_m, V_m(\C^3))$ is a complex representation of $\SU{3}$.}\label{part:3a}
\part{Find the weights for $\pi_1$ and $\pi_2$, the $\slie{3}{\C}$-representations associated to $\Pi_1$ and $\Pi_2$, respectively.}\label{part:3b}
\part{Prove that $(\pi_1, V_1(\C^3))$ and $(\pi_2, V_2(\C^3))$ are irreducible representations of $\slie{3}{\C}$. What are their highest weights?}\label{part:3c}
\end{parts}
\end{problem}
\begin{solution}
\ref{part:3a}
\begin{equation*}
\Pi_m(A)\qty(\qty\Big[\Pi_m(B)f])(z) = \qty\Big[\Pi_m(B)f](A^{-1}z) = f(B^{-1}A^{-1}z) = \qty\Big[\Pi_m(AB)f](z)
\end{equation*}
\ref{part:3b}
The action of an arbitrary element $X\in\slie{3}{\C}$ under the representation $\pi_m$ is given by
\begin{align*}
\pi_m(X) & = -\qty(X_{11} z_1 + X_{12}z_2 + X_{13}z_3)\pdv{z_1} \\
& \quad -\qty(X_{21}z_1 + X_{22}z_2 + X_{23}z_3)\pdv{z_2} \\
& \quad -\qty(X_{31}z_1 + X_{32}z_2 + X_{33}x_3)\pdv{z_3}
\end{align*}
Thus, for $H_1$ and $H_2$ we have
\begin{align*}
\pi_m(H_1) & = z_2\pdv{z_2} - z_1\pdv{z_1} \\
\pi_m(H_2) & = z_3\pdv{z_3} - z_2\pdv{z_2}
\end{align*}
Take $m = 1$ where $V_1 = \vspan_\C\qty{z_1, z_2, z_3}$. Applying $\pi_1(H_1)$ and $\pi_1(H_2)$ to an arbitrary element $f = az_1 + bz_2 + cz_3$ and ensuring it is an eigenvector yields the following two equations:
\begin{align*}
(m_1 + 1)az_1 + (m_1 - 1)bz_2 + cm_1z_3 & = 0 \\
m_2az_1 + (m_2 + 1)bz_2 + (m_2 - 1)cz_3 & = 0
\end{align*}
From here we can see there are three weights possible tabulated in \cref{tab:pione}.
\begin{table}[h]
\centering\begin{tabular}{c c c}
Weight & Eigenvector & Multiplicity \\ \toprule
$(1, -1)$ & $bz_2$ & 1 \\
$(-1, 0)$ & $az_1$ & 1 \\
$(0, 1)$ & $cz_3$ & 1
\end{tabular}
\caption{Weight Decomposition for $(\pi_1, V_1(\C^3))$}\label{tab:pione}
\end{table}
Take $m = 2$ where $V_2 = \vspan_\C\qty{z_1^2, z_2^2, z_3^2, z_1z_1, z_1z_3, z_2z_3}$ and we can repeat the process as above with an arbitrary element $f = az_1^2 + bz_2^2 + cz_3^2 + dz_1z_2 + ez_1z_3 + gz_2z_3$.
\begin{align*}
\pi_2(H_1)f & = -2az_1^2 + 2bz_2^2 - ez_1z_3 + gz_2z_3 \\
\pi_2(H_2)f & = -2bz_2^2 + 2cz_3^2 - dz_1z_d + ez_1z_3
\end{align*}
From here we can read off the weights and eigenvectors, probably much easier than the equation I wrote down for the $m = 1$ case.
\begin{table}[h]
\centering\begin{tabular}{c c c}
Weight & Eigenvector & Multiplicity \\ \toprule
$(-2, 0)$ & $az_1^2$ & 1 \\
$(-2, 2)$ & $bz_2^2$ & 1 \\
$(0, 2)$ & $cz_3^2$ & 1 \\
$(0, -1)$ & $dz_1z_1$ & 1 \\
$(-1, 0)$ & $ez_1z_3$ & 1 \\
$(1, 0)$ & $gz_2z_3$ & 1
\end{tabular}
\caption{Weight Decomposition for $(\pi_2, V_2(\C^3))$}\label{tab:pitwo}
\end{table}
\ref{part:3c}
To show $(\pi_1, V_1(\C^3))$ and $(\pi_2, V_2(\C^3))$ are irreps we will first show they are highest weight cyclic representations. Then using Proposition 6.14 from Hall, and the fact that all representations of $\slie{3}{\C}$ are completely reducible, we can deduce that the aforementioned representations are irreducible.
For the $m = 1$ case we have highest weight vector $v = cz_3$ with weight $(0, 1)$. This is easily verified (although tedious) by computing $\mu_i - \mu_j = a\alpha_1 + b\alpha_2$ for the weights in \cref{tab:pione}. Thus condition 1 is satisfied. Now we can apply each $X_i$ to $v$ to see if it's annihilated.
\begin{align*}
\pi_1(X_1)v & = -z_2\pdv{z_1}\qty(cz_3) = 0 \\
\pi_1(X_2)v & = -z_3\pdv{z_2}\qty(cz_3) = 0 \\
\pi_1(X_2)v & = -z_3\pdv{z_1}\qty(cz_3) = 0
\end{align*}
Thus we also have condition two that $\pi_1(X_i)v = 0$. Lastly we have to verify $V_1(\C^3)$ is the smallest invariant subspace that contains $v$. We can do this by creating the ``tree'' applying all $\pi_1(Y_i)$. We use the convention of ``left'' means apply $\pi_1(Y_1)$, ``center'' means $\pi_1(Y_2)$ and ``right'' means $\pi_1(Y_3)$.
\Tree[.$z_3$ [.0 ]
[.$z_2$ [.$z_1$ ]
[.0 ]
[.0 ]]
[.$z_1$ [.0 ]
[.0 ]
[.0 ]]]
This diagram shows there no invariant subspace containing $v$ that is not the entirety of $V_1(\C^3)$. Thus $(\pi_1, V_1(\C^3))$ is a cyclic representation with highest weight $(0, 1)$ and by the argument given at the outset of~\ref{part:3c} we have an irrep.
Now take $m = 2$ and we will run through the same process. The highest weight in \cref{tab:pitwo} is $(0, 2)$ again by (tediously) computing $\mu_i - \mu_j = a\alpha_1 + b\alpha_2$ repeatedly. We can now check if $v = cz_3^2$ is annihilated by all $\pi_2(X_i)$.
\begin{align*}
\pi_2(X_1)v & = -z_2\pdv{z_1}\qty(cz_3^2) = 0 \\
\pi_2(X_2)v & = -z_3\pdv{z_2}\qty(cz_3^2) = 0 \\
\pi_2(X_2)v & = -z_3\pdv{z_1}\qty(cz_3^2) = 0
\end{align*}
And again now we need to check if there is a smaller invariant subspace containing $v$.
\Tree[.$z_3^2$ [.0 ]
[.$z_2z_3$ [.$z_1z_3$ [.0 ] [.$z_1z_2$ ] [.$z_1^2$ [.0 ] [.0 ] [.0 ]]]
[.$z_2^2$ [.$z_1z_2$ ] [.0 ] [.0 ]]
[.$z_1z_2$ ]]
[.$z_1z_3$ [.0 ]
[.$z_1z_2$ [.$z_1^2$ ] [.0 ] [.0 ]]
[.$z_1z_3$ ]]]
So indeed this representation is highest weight cyclic with weight $(0, 2)$ and is thus irreducible by the above logic.
\end{solution}
\begin{problem}
In each part below, verify that $\mfr{t}$ is a Cartan subalgebra of $\mfr{g} = \operatorname{Lie}(G)$. Then find the maximal torus in $G$ corresponding to $\mfr{t}$.
\begin{parts}
\part{$G = \SO{2n}$; $\mfr{t} = \qty{ \smqty(0 & \theta_1 & & & \\ -\theta_1 & 0 & & & \\ & & \ddots & & \\ & & & 0 & \theta_n\\ & & & -\theta_n & 0) : \theta_i \in \R }$.}\label{part:4a}
\part{$G = \SO{2n + 1}$; $\mfr{t} = \qty{ \smqty(0 & \theta_1 & & & & \\ -\theta_1 & 0 & & & & \\ & & \ddots & & & \\ & & & 0 & \theta_n & \\ & & & -\theta_n & 0 & \\ & & & & & 0 ) : \theta_i \in \R }$.}\label{part:4b}
\end{parts}
\end{problem}
\begin{solution}
\ref{part:4a}
First lets verify $\mfr{t}$ is indeed a Cartan subalgebra. The Lie algebra $\so{2n}$ consists of $2n\times 2n$ skew-symmetric matrices, which clearly $\mfr{t}$ is a subset of. In order to show it's a subalgebra, it must be closed under the commutator, but because this is a \emph{Cartan} subalgebra we have the extra condition that $\comm{X}{Y} = 0$ for all $X, Y\in \mfr{t}$. We'll write elements in $\mfr{t}$ in block form using $R_\alpha = \smqty[0 & \alpha \\ -\alpha & 0]$.
\begin{align*}
\comm{\mqty(\dmat{R_{\theta_1}, \ddots, R_{\theta_n}})}{\mqty(\dmat{R_{\phi_1}, \ddots, R_{\phi_n}})} & = \smqty(\dmat{R_{\theta_1}R_{\phi_1} - R_{\phi_1}R_{\theta_1}, \ddots, R_{\theta_n}R_{\phi_n} - R_{\phi_n}R_{\theta_n}})
\end{align*}
Now to calculate the terms on the diagonal:
\begin{align*}
R_{\theta_i}R_{\phi_i} - R_{\phi_i}R_{\theta_i} & = \mqty(0 & \theta_i \\ -\theta_i & 0 )\mqty(0 & \phi_i \\ -\phi_i & 0) - \mqty(0 & \phi_i \\ -\phi_i & 0 )\mqty(0 & \theta_i \\ -\theta_i & 0) \\
& = \mqty(-\theta_i\phi_i & 0 \\ 0 & -\theta_i\phi_i) - \mqty(-\theta_i\phi_i & 0 \\ 0 & -\theta_i\phi_i) \\
& = \mqty(0 & 0 \\ 0 & 0)
\end{align*}
Thus everything in $\mfr{t}$ commutes, and is also closed under the bracket/commutator since the zero matrix is skew symmetric.
Now we must show that anything that commutes with \emph{every} element of $\mfr{t}$ is also in $\mfr{t}$. That is suppose we have some $X\in\so{2n}$ such that $\comm{X}{\mfr{t}} = 0$. Writing things out in coordinates for $C = XA$ and $D = AX$ we have
\begin{align*}
C_{ij} & = \sum_{k = 1}^{2n}X_{ik}A_{kj} = X_{i,j + 1}A_{j + 1, j} = -\theta_j X_{i, j + 1} \\
D_{ij} & = \sum_{k = 1}^{2n}A_{ik}X_{kj} = A_{i, i + 1}X_{i + 1,j} = \theta_j X_{i + 1, j}
\end{align*}
And these must be equal, so we have
\begin{equation}\label{eq:antisym}
\theta_i X_{i +1, j} + \theta_j X_{i, j + 1} = 0.
\end{equation}
When $i = j$ then $X_{i+1, i} + X_{i, i+1} = 0$, which $A$ also satisfies. Since~\cref{eq:antisym} must be satisfied for all $X\in\mfr{t}$, it must be satisfied for $X$ such that $\theta_i = 0$ for all $i\in\Z_n$ except for one $j$ where $\theta_j = 1$. Plugging these into~\cref{eq:antisym} we see $X_{i, j+1} = 0$ for all $i\neq j$. This, combined with the fact that $X\in\so{2n}$ is anti-symmetric shows that $X\in\mfr{t}$.
Now let's compute the maximal torus corresponding to $\mfr{t}$. It'll be helpful to compute the first few powers of an element of $\mfr{t}$ to get a sense of what's going on.
\begin{align*}
A^2 & = \mqty(0 & \theta_1 & & & \\ -\theta_1 & 0 & & & \\ & & \ddots & & \\ & & & 0 & \theta_n\\ & & & -\theta_n & 0)^{\!\!\!2} = \mqty(-\theta_1^1 & & & & \\ & -\theta_1^2 & & & \\ & & \ddots & & \\ & & & -\theta_n^2 & \\ & & & & -\theta_n^2) \\
A^3 & = \mqty(0 & \theta_1 & & & \\ -\theta_1 & 0 & & & \\ & & \ddots & & \\ & & & 0 & \theta_n\\ & & & -\theta_n & 0)^{\!\!\!3} = \mqty(0 & -\theta_1^3 & & & \\ \theta_1^3 & 0 & & & \\ & & \ddots & & \\ & & & 0 & -\theta_n^3 \\ & & & \theta_n^3 & 0)
\end{align*}
These give a pretty good hint what the next terms are. Hence we can write
\begin{align*}
\e^{A} & = \1 + A + A^2 + A^3 + \cdots \\
& = \mqty(1 - \theta_1^2 + \theta_1^4 + \cdots & \theta_1 - \theta_1^3 + \theta_1^5 - \cdots & \\ -\theta_1 + \theta_1^3 - \theta_1^5 + \cdots & 1 - \theta_1^2 + \theta_1^4 + \cdots & \\ & & \ddots) \\
& = \mqty(\cos\theta_1 & \sin\theta_1 & & & \\ -\sin\theta_1 & \cos\theta_1 & & \\ & & \ddots & & \\ & & & \cos\theta_n & \sin\theta_n \\ & & & -\sin\theta_n & \cos\theta_n)
\end{align*}
Thus maximal torus in $\SO{2n}$ is (up to isomorphism) $\diag(R_{\theta_1},\ldots,R_{\theta_n})$ where $R_\alpha = \smqty[\phantom{-}\cos\alpha & \sin\alpha \\ -\sin\alpha & \cos\alpha]$.
\ref{part:4b}
The computations performed above are idential for this case, where there is an additional row and column of 0's to work with. Thus the maximal torus of $\SO{2n + 1}$ is $\diag(R_{\theta_1},\ldots,R_{\theta_n}, \1_{2\times 2})$ which is easily seen to be isomorphic to that of $\SO{2n}$. The $\1_{2\times 2}$ arises from the first term of $\e^{A} = \1 + \cdots$.
\end{solution}
\begin{problem}
\begin{parts}
\part{Let $n \geq 3$ and let $H$ be the set of diagonal matrices in $\SO{n}$. Prove that $H$ is a maximal closed abelian subgroup of $\SO{n}$, but is not contained in any maximal torus.}\label{part:5a}
\part{By contrast, let $H$ be any closed abelian subgroup of $\SU{n}$. Prove that $H$ is contained in a maximal torus.}\label{part:5b}
\end{parts}
\end{problem}
\begin{solution}
\ref{part:5a}
Note that $H$ is the the collection of matrices of the form $\diag(\pm 1, \ldots, \pm 1)$ with an even number of $-1$'s on the diagonal. This can be seen from the maximal torus of $\SO{n}$ shown in the previous problem.
First we need to show $H$ is a maximal abelian subgroup of $\SO{n}$. Suppose $A\in\SO{n}$ commutes with all $B\in H$. We can write $A$ in canonical form as
\begin{equation*}
A = \diag(R_1,\ldots, R_k, \pm 1, \ldots, \pm 1)
\end{equation*}
where there are an even number of $-1$'s and 0's everywhere else. Using the fact that commuting matrices preserve each others' eigenspaces we see $A$ must preserve the eigenspaces of $B$. Since every standard basis vector $\vb{e}_i\in\R^n$ is an eigenvector of $B$, $A$ must map each $A\vb{e}_i = \lambda_i\vb{e}_i$. Thus all the $2\times 2$ block matrices must be plus or minus 1's. Thus $B\in H$.
\ref{part:5b}
Since all $X, Y\in H$ commute, they can be simultaneously diagonalized in some basis. The eigenvalues of a unitary matrix are all unit complex numbers, and hence any element in $H$ can be written as
\begin{equation*}
X = \diag(\e^{\iu\alpha_1},\ldots, \e^{\iu\alpha_n}).
\end{equation*}
Since the determinant of $X$ is 1, we have the condition that $\prod_i\e^{\iu\alpha_i} = 1$ which restricts one\footnote{And can be made to be the last.} $\alpha_i$ so we have
\begin{equation*}
X = \diag(\e^{\iu\alpha_1},\ldots, \e^{-\iu \sum_{i = 1}^{n-1}\alpha_i}).
\end{equation*}
So every $X\in H$ can be specified by $n-1$ unit complex numbers. Thus $H$ is clearly contained in the maximal torus of $\SU{n}$.
\end{solution}
\begin{problem}
Let $T$ be the set of diagonal matrices in $\U{n}$ and $W$ its Weyl group. Let $S_n$ be the permutation group of $\qty{1, \ldots, n}$ and define an action of $S_n$ on $T$ by
\[
\sigma \cdot' \diag(u_1, \ldots, u_n) = \diag(u_{\sigma^{-1}(1)}, \ldots, u_{\sigma^{-1}(n)}).
\]
(Here we put a prime in the notation to distinguish this action from the action of $W$ on $T$.) Also, take a generating element $t_0 = \diag(\e^{2\pi\iu \theta_1}, \ldots, \e^{2\pi\iu \theta_n})$ in $T$.
\begin{parts}
\part{Given $w \in W$, prove that there exists a unique $\sigma \in S_n$ such that
\[
w \cdot t_0 = \sigma \cdot' t_0.
\]
Deduce that $w \cdot t = \sigma \cdot' t$ for all $t \in T$.}\label{part:6a}
\part{In the notation of part (a), prove that the map $w \mapsto \sigma$ defines an injective homomorphism from $W$ into $S_n$.}\label{part:6b}
\part{Prove that the homomorphism in part (b) is also surjective. (Consequently, $W$ is isomorphic to $S_n$.)}\label{part:6c}
\end{parts}
\end{problem}
\begin{solution}
\ref{part:6a}
We have $w\cdot t_0 = xt_0x^{-1} = t'\in T$ and because $t$ and $t'$ only differ by conjugation, they must have the same spectrum, however it's possibly ``rearranged''. This can clearly be done by $\sigma\cdot't_0$, but we need to show it's unique. Suppose we have $\sigma, \tilde{\sigma}\in S_n$ such that $w\cdot t_0 = \sigma\cdot't_0 = \tilde{\sigma}\cdot't_0$. Thus we have
\begin{equation*}
\diag(\e^{2\pi\iu\theta_{\sigma^{-1}(1)}}, \ldots, \e^{2\pi\iu\theta_{\sigma^{-1}(n)}}) = \diag(\e^{2\pi\iu\theta_{\tilde{\sigma}^{-1}(1)}}, \ldots, \e^{2\pi\iu\theta_{\tilde{\sigma}^{-1}(n)}})
\end{equation*}
and these must be componentwise equal so
\begin{equation*}
\e^{2\pi\iu\theta_{\sigma^{-1}(i)}} = \e^{2\pi\iu\theta_{\tilde{\sigma}^{-1}(i)}}.
\end{equation*}
This implies $\theta_{\sigma^{-1}(i)} = \theta_{\tilde{\sigma}^{-1}(i)} + n$ for some $n\in \Z$, but by the linear independence\footnote{over $\Q$} of $1$ and the $\theta_i$'s, this is only possible if $\sigma^{-1} = \tilde{\sigma}^{-1}$ and thus $n = 0$, and by the bijectivity of elements in $S_n$, $\sigma = \tilde{\sigma}$.
Since $t_0$ generates, we can always write $t = \lim_{n\to\infty}t_0^{a_n}$ for some subsequence $a_n$ of $\Z$. We then have
\begin{equation*}
w\cdot t = x\qty[\lim_{n\to\infty}t_0^{a_n}]x^{-1} = \lim_{n\to\infty}xt_0^{a_n}x^{-1} = \lim_{n\to\infty}\qty[\sigma\cdot't_0]^n = \sigma\cdot' t
\end{equation*}
\ref{part:6b}
Let $f: W\to S_n$ be the map such that $w\mapsto \sigma$.
This map is indeed a homomorphism:
\begin{equation*}
(w_1w_2)\cdot t = x_1x_2tx_2^{-1}x_1^{-1} = x_2(\sigma_2\cdot't)x_2^{-1} = (\sigma_1\circ \sigma_2)\cdot't
\end{equation*}
To show this map is an injection, suppose $w_1\mapsto \sigma$ and $w_2\mapsto\sigma$, that is $f(w_1) = f(w_2)$, where they can only be seen as equal if they are equal on all inputs.
\begin{align*}
f(w_1)\cdot't & = f(w_2)\cdot't \\
\sigma\cdot't & = \sigma\cdot't \\
x_1tx_1^{-1} & = x_2tx_2^{-1} \\
\qty(x_2^{-1}x_1)t & = t\qty(x_2^{-1}x_1)
\end{align*}
Thus $x_2^{-1}x_1$ commutes with all $t$, and hence is in $T$. This implies $x_2^{-1}x_1$ is modded out of the Weyl group and equals the identity in $W$. Hence $w_1 = w_2$.
\ref{part:6c}
To see that $f$ is surjective, note that we can construct a basis change that swaps any basis vectors around in an element $x$ which we conjugate by in $w\cdot t$. Since there are $n!$ ways to swap around basis vectors, we can surely hit every element of $S_n$.\footnote{I know this is sloppy, but I'm \emph{tired}, and I'm not sure what it is, but the Weyl group doesn't \emph{feel} cool.}
\end{solution}
\begin{problem}
Let $G$ be a compact connected matrix Lie group.
\begin{parts}
\part{Let $f: G \to H$ be a surjective Lie group homomorphism from $G$ onto another compact connected matrix Lie group. Prove that if $T$ is a maximal torus in $G$ then $f(T)$ is a maximal torus in $H$. Deduce that if $H$ is abelian then the restriction $f|_{T}$ is surjective already.}\label{part:8a}
\part{Given $g \in G$ and $n \in \N$, prove that there exists $h \in G$ such that $h^n = g$.}\label{part:8b}
\end{parts}
\end{problem}
\begin{solution}
\ref{part:8a}
Since $T$ is connected and compact, and $f$ is a continuous function, $f(T)$ is also connected and compact. Since $f$ is a homomorphism we have $f(a)f(b) = f(ab) = f(ba) = f(b)f(a)$ if $a, b\in T$, and thus $f(T)$ is also commutative and by Theorem 11.2 in Hall, $f(T)$ is a torus.
To show $f(T)$ is maximal in $H$, take $K\subseteq H$ to be a torus containing $f(T)$. Take an element $h\in K$, and by the surjectivity of $f$ we are guaranteed to be able to find a $g\in G$ such that $f(g) = h$. Since we can write $g = xtx^{-1}$ by Lemma 11.12 in Hall, we have
\begin{equation*}
h = f(g) = f(xtx^{-1}) = \underbrace{f(x)}_{\in H}\underbrace{f(t)}_{\in f(T)}f(x)^{-1}.
\end{equation*}
That is, any element $h\in K$ can be decomposed as $h = f(x)f(t)f(x)^{-1}$ which implies $K = yf(T)y^{-1}$, and thus $f(T)$ is maximal.
\ref{part:8b}
Since $G$ is compact and connected, we know exp: $\mathsf{g}\to G$ is surjective, and hence for all $g\in G$ we can fine an $A\in\mathsf{g}$ such that $g = \e^{A}$. In particular we can also define $h\defeq \e^{A/n}$ so that $h^n = g$.
\end{solution}
\end{document}